问题

分析 >> 数学分析
Questions in category: 数学分析 (Mathematical Analysis).

证明: $(\cos\theta)^p\leqslant\cos(p\theta)$, 这里 $\theta\in[0,2\pi]$, $0 < p <1$.

Posted by haifeng on 2016-10-19 13:30:45 last update 2016-10-19 13:30:45 | Answers (1) | 收藏


证明: $(\cos\theta)^p\leqslant\cos(p\theta)$, 这里 $\theta\in[0,2\pi]$, $0 < p <1$.

 


Berkeley Problems in Mathematics, Third Edition. (伯克利数学问题集)